Những câu hỏi liên quan
Tiến Nguyễn Minh
Xem chi tiết
✰๖ۣۜŠɦαɗøω✰
27 tháng 3 2020 lúc 8:31

Bài 1 : 

Phương trình <=> 2x . x2 = ( 3y + 1 ) + 15

Vì \(\hept{\begin{cases}3y+1\equiv1\left(mod3\right)\\15\equiv0\left(mod3\right)\end{cases}\Rightarrow\left(3y+1\right)^2+15\equiv1\left(mod3\right)}\)

\(\Rightarrow2^x.x^2\equiv1\left(mod3\right)\Rightarrow x^2\equiv1\left(mod3\right)\)

( Vì số  chính phương chia 3 dư 0 hoặc 1 ) 

\(\Rightarrow2^x\equiv1\left(mod3\right)\Rightarrow x\equiv2k\left(k\inℕ\right)\)

Vậy \(2^{2k}.\left(2k\right)^2-\left(3y+1\right)^2=15\Leftrightarrow\left(2^k.2.k-3y-1\right).\left(2^k.2k+3y+1\right)=15\)

Vì y ,k \(\inℕ\)nên 2k . 2k + 3y + 1 > 2k .2k - 3y-1>0

Vậy ta có các trường hợp: 

\(+\hept{\begin{cases}2k.2k-3y-1=1\\2k.2k+3y+1=15\end{cases}\Leftrightarrow\hept{\begin{cases}2k.2k=8\\3y+1=7\end{cases}\Rightarrow}k\notinℕ\left(L\right)}\)

\(+,\hept{\begin{cases}2k.2k-3y-1=3\\2k.2k+3y+1=5\end{cases}\Leftrightarrow\hept{\begin{cases}2k.2k=4\\3y+1=1\end{cases}\Rightarrow}\hept{\begin{cases}k=1\\y=0\end{cases}\left(TM\right)}}\)

Vậy ( x ; y ) =( 2 ; 0 ) 

Bình luận (0)
 Khách vãng lai đã xóa
Phùng Gia Bảo
27 tháng 3 2020 lúc 9:14

Bài 3: 

Giả sử \(5^p-2^p=a^m\)    \(\left(a;m\inℕ,a,m\ge2\right)\)

Với \(p=2\Rightarrow a^m=21\left(l\right)\)

Với \(p=3\Rightarrow a^m=117\left(l\right)\)

Với \(p>3\)nên p lẻ, ta có

\(5^p-2^p=3\left(5^{p-1}+2.5^{p-2}+...+2^{p-1}\right)\Rightarrow5^p-2^p=3^k\left(1\right)\)    \(\left(k\inℕ,k\ge2\right)\)

Mà \(5\equiv2\left(mod3\right)\Rightarrow5^x.2^{p-1-x}\equiv2^{p-1}\left(mod3\right),x=\overline{1,p-1}\)

\(\Rightarrow5^{p-1}+2.5^{p-2}+...+2^{p-1}\equiv p.2^{p-1}\left(mod3\right)\)

Vì p và \(2^{p-1}\)không chia hết cho 3 nên \(5^{p-1}+2.5^{p-2}+...+2^{p-1}⋮̸3\)

Do đó: \(5^p-2^p\ne3^k\), mâu thuẫn với (1). Suy ra giả sử là điều vô lý

\(\rightarrowĐPCM\)

Bình luận (0)
 Khách vãng lai đã xóa
Lê Nhật Khôi
27 tháng 3 2020 lúc 10:53

Bài 4:

Ta đặt: \(S=6^m+2^n+2\)

TH1: n chẵn thì:

\(S=6^m+2^n+2=6^m+2\left(2^{n-1}+1\right)\)

Mà \(2^{n-1}+1⋮3\Rightarrow2\left(2^{n-1}+1\right)⋮6\Rightarrow S⋮6\)

Đồng thời S là scp

Cho nên: \(S=6^m+2\left(2^{n-1}\right)=\left(6k\right)^2\)

\(\Leftrightarrow6^m+6\left(2^{n-2}-2^{n-3}+...+2-1\right)=36k^2\)

Đặt: \(A\left(n\right)=2^{n-2}-2^{n-3}+...+2-1=2^{n-3}+...+1\)là số lẻ

Tiếp tục tương đương: \(6^{m-1}+A\left(n\right)=6k^2\)

Vì A(n) lẻ và 6k^2 là chẵn nên: \(6^{m-1}\)lẻ\(\Rightarrow m=1\)

Thế vào ban đầu: \(S=8+2^n=36k^2\)

Vì n=2x(do n chẵn) nên tiếp tục tương đương: \(8+\left(2^x\right)^2=36k^2\)

\(\Leftrightarrow8=\left(6k-2^x\right)\left(6k+2^x\right)\)

\(\Leftrightarrow2=\left(3k-2^{x-1}\right)\left(3k+2^{x-1}\right)\)

Vì \(3k+2^{x-1}>3k-2^{x-1}>0\)(lớn hơn 0 vì 2>0 và \(3k+2^{x-1}>0\))

Nên: \(\hept{\begin{cases}3k+2^{x-1}=2\\3k-2^{x-1}=1\end{cases}}\Leftrightarrow6k=3\Rightarrow k\notin Z\)(loại)

TH2: n là số lẻ

\(S=6^m+2^n+2=\left(2k\right)^2\)(do S chia hết cho 2 và S là scp)

\(\Leftrightarrow3\cdot6^{m-1}+2^{n-1}+1=2k^2\)là số chẵn

\(\Rightarrow3\cdot6^{m-1}+2^{n-1}\)là số lẻ

Chia tiếp thành 2TH nhỏ: 

TH2/1: \(3\cdot6^{m-1}\)lẻ và \(2^{n-1}\)chẵn với n là số lẻ

Ta thu đc: m=1 và thế vào ban đầu

\(S=2^n+8=\left(2k\right)^2\)(n lớn hơn hoặc bằng 3)

\(\Leftrightarrow2^{n-2}+2=k^2\)

Vì \(k^2⋮2\Rightarrow k⋮2\Rightarrow k^2=\left(2t\right)^2\)

Tiếp tục tương đương: \(2^{n-2}+2=4t^2\)

\(\Leftrightarrow2^{n-3}+1=2t^2\)

\(\Leftrightarrow2^{n-3}\)là số lẻ nên n=3

Vậy ta nhận đc: \(\left(m;n\right)=\left(1;3\right)\)

TH2/2: \(3\cdot6^{m-1}\)là số chẵn và \(2^{n-1}\)là số lẻ

Suy ra: n=1

Thế vào trên: \(6^m+4=4k^2\)

\(\Leftrightarrow6^m=\left(2k-2\right)\left(2k+2\right)\)

\(\Leftrightarrow\hept{\begin{cases}2k-2=6^q\\2k+2=6^p\end{cases}}\Rightarrow p+q=m\)

Và \(6^p-6^q=4\)

\(\Leftrightarrow6^q\left(6^{p-q}-1\right)=4\Leftrightarrow6^q\le4\Rightarrow q=1\)(do là tích 2 stn)

\(\Rightarrow k\notin Z\)

Vậy \(\left(m;n\right)=\left(1;3\right)\)

P/S: mk không kiểm lại nên có thể sai

Bình luận (0)
 Khách vãng lai đã xóa
Lê Thành An
Xem chi tiết
Hoàng Nguyễn Văn
3 tháng 1 2020 lúc 23:35

Ta có \(\left(x+y\right)^3=\left(x-y-6\right)^2\left(1\right)\)

Vì x,y nguyên dương nên

\(\left(x+y\right)^3>\left(x+y\right)^2\)kết hợp (1) ta được:

\(\left(x-y-6\right)^2>\left(x+y\right)^2\Leftrightarrow\left(x+y\right)^2-\left(x-y-6\right)^2< 0\Leftrightarrow\left(x-3\right)\left(y+3\right)< 0\)

Mà y+3 >0 (do y>0)\(\Rightarrow x-3< 0\Leftrightarrow x< 3\)

mà \(x\inℤ^+\)\(\Rightarrow x\in\left\{1;2\right\}\)

*x=1 thay vào (1) ta có:

\(\left(1+y\right)^3=\left(1-y-6\right)^2\Leftrightarrow y^3+3y^2+3y+1=y^2+10y+25\Leftrightarrow\left(y-3\right)\left(y^2+5y+8\right)=0\)

mà \(y^2+5y+8=\left(y+\frac{5}{2}\right)^2+\frac{7}{4}\ge\frac{7}{4}>0\)

\(\Rightarrow y-3=0\Leftrightarrow y=3\inℤ^+\)

*y=2 thay vào (1) ta được: 

\(\left(2+y\right)^3=\left(2-y-6\right)^2\Leftrightarrow y^3+6y^2+12y+8=y^2+8y+16\Leftrightarrow y^3+5y^2+4y-8=0\)

Sau đó cm pt trên không có nghiệm nguyên dương.

Vậy x=1;y=3

Bình luận (0)
 Khách vãng lai đã xóa
VUX NA
Xem chi tiết
Kill Myself
Xem chi tiết
Công Tử Họ Nguyễn
9 tháng 10 2018 lúc 21:27

Giả sử (x;y) là cặp số nguyên dương cần tìm. Khi đó ta có: 
(xy-1) I (x^3+x) => (xy-1) I x.(x^2+1) (1) 
Do (x; xy-1) =1 ( Thật vậy: gọi (x;xy-1) =d => d I x => d I xy => d I 1). 
Nên từ (1) ta có: 
(xy-1) I (x^2+1) 
=> (xy-1) I (x^2+1+xy -1) => (xy-1) I (x^2+xy) => (xy-1) I x.(x+y) => (xy-1) I (x+y) 
Điều đó có nghĩa là tồn tại z ∈ N* sao cho: 
x+y = z(xy-1) <=> x+y+z =xyz (2) 

[Đây lại có vẻ là 1 bài toán khác] 
Do vai trò bình đẳng nên ta giả sử: x ≥ y ≥ z. 
Từ (2) ta có: x+y+z ≤ 3x => 3x ≥ xyz => 3 ≥ yz ≥ z^2 => z=1 
=> 3 ≥ y => y ∈ {1;2;3} 
Nếu y=1: x+2 =x (loại) 
Nếu y=2: (2) trở thành x+3 =2x => x=3 
Nếu y=3: x+4 = 3x => x=2 (loại vì ta có x≥y) 
Vậy khi x ≥ y ≥ z thì (2) có 1 nghiệm (x;y;z) là (3;2;1) 
Hoán vị vòng quanh được 6 nghiệm là: .....[bạn tự viết nhé] 

Vậy bài toán đã cho có 6 nghiệm (x;y) là : .... [viết y chang nhưng bỏ z đi]

Bình luận (0)
Kim
9 tháng 10 2018 lúc 21:27

 Giả sử (x;y) là cặp số nguyên dương cần tìm. Khi đó ta có: 
(xy-1) I (x^3+x) => (xy-1) I x.(x^2+1) (1) 
Do (x; xy-1) =1 ( Thật vậy: gọi (x;xy-1) =d => d I x => d I xy => d I 1).
Nên từ (1) ta có: 
(xy-1) I (x^2+1) 
=> (xy-1) I (x^2+1+xy -1) => (xy-1) I (x^2+xy) => (xy-1) I x.(x+y) => (xy-1) I (x+y) 
Điều đó có nghĩa là tồn tại z ∈ N* sao cho: 
x+y = z(xy-1) <=> x+y+z =xyz (2) 

[Đây lại có vẻ là 1 bài toán khác] 
Do vai trò bình đẳng nên ta giả sử: x ≥ y ≥ z. 
Từ (2) ta có: x+y+z ≤ 3x => 3x ≥ xyz => 3 ≥ yz ≥ z^2 => z=1 
=> 3 ≥ y => y ∈ {1;2;3} 
Nếu y=1: x+2 =x (loại) 
Nếu y=2: (2) trở thành x+3 =2x => x=3 
Nếu y=3: x+4 = 3x => x=2 (loại vì ta có x≥y) 
Vậy khi x ≥ y ≥ z thì (2) có 1 nghiệm (x;y;z) là (3;2;1) 
Hoán vị vòng quanh được 6 nghiệm là: .....[bạn tự viết nhé] 

Vậy bài toán đã cho có 6 nghiệm (x;y) là : .... [viết y chang nhưng bỏ z đi]

Bình luận (0)
Hoàng Thế Hải
9 tháng 10 2018 lúc 21:34

Xét x= 1 => \(\dfrac{2}{y-1}\in\mathbb N\), từ đó có \(y=2\vee y=3\)

Xét y=1 => \(\dfrac{x^3+x}{x-1}=x^2+x+2+\dfrac{2}{x-1}\in\mathbb N\), từ đó có \(x=2\vee x=3\)

Xét \(x\ge 2\) hoặc \(y\ge 2\) . Ta có : \((x,xy-1)=1\). Do đó :

\(xy-1|x^3+x\Rightarrow xy-1|x^2+1\Rightarrow xy-1|x+y\)

=> \(x+y\ge xy-1\Rightarrow (x-1)(y-1)\le 2\). Từ đó có \((x-1)(y-1)=1\ \vee (x-1)(y-1)=2\) 

=> x = y = 2 ( loại ) hoặc x = 2 ; y = 3 hoặc x = 3 ; y= 2

Vậy các cặp số ( x;y ) thỏa mãn là (1;2),(2;1),(1;3),(3;1),(2;3),(3;2)

Bình luận (0)
VUX NA
Xem chi tiết
Yeutoanhoc
26 tháng 8 2021 lúc 15:04

Sao cho gì vậy bạn ?

Bình luận (0)
VUX NA
26 tháng 8 2021 lúc 15:05

là số nguyên

 

Bình luận (0)
ミ★ήɠọς τɾίếτ★彡
26 tháng 8 2021 lúc 15:05

đề có phải là:Tìm tất cả các cặp số nguyên dương (a;b) sao cho\(\dfrac{ab\left(a+b\right)}{ab+2}\) là số nguyên không bạn

Bình luận (1)
Bách Out Trình
Xem chi tiết
๖ۣۜDũ๖ۣۜN๖ۣۜG
6 tháng 2 2022 lúc 0:41

\(x^3+y^3-9xy=0\)

\(\Leftrightarrow\left(x+y\right)^3-3x^2y-3xy^2-9xy=0\)

\(\Leftrightarrow\left(x+y\right)^3+27-3xy\left(x+y+3\right)=27\)

\(\Leftrightarrow\left(x+y+3\right)\left[\left(x+y\right)^2-3\left(x+y\right)+9\right]-3xy\left(x+y+3\right)-27=0\)

\(\Leftrightarrow\left(x+y+3\right)\left(x^2+2xy+y^2-3x-3y+9-3xy\right)-27=0\)

\(\Leftrightarrow\left(x+y+3\right)\left(x^2-xy+y^2-3x-3y+9\right)-27=0\)

\(\Leftrightarrow\left(x+y+3\right)\left(2x^2-2xy+2y^2-6x-6y+18\right)-54=0\)

\(\Leftrightarrow\left(x+y+3\right)\left[\left(x-y\right)^2+\left(x-3\right)^2+\left(y-3\right)^2\right]=54\)

Do x, y > 0 => x + y + 3 > 3

Mà x, y nguyên dương => \(\left\{{}\begin{matrix}x+y+3\in Z^+\\\left(x-y\right)^2+\left(x-3\right)^2+\left(y-3\right)^2\in Z^+\end{matrix}\right.\)

Và \(\left(x-y\right)^2+\left(x-3\right)^2+\left(y-3\right)^2⋮2\)

TH1: \(\left\{{}\begin{matrix}x+y+3=9\\\left(x-y\right)^2+\left(x-3\right)^2+\left(y-3\right)^2=6\end{matrix}\right.\)

\(\Leftrightarrow\left\{{}\begin{matrix}x+y=6\\x^2-xy+y^2-3x-3y=-6\end{matrix}\right.\)

\(\Leftrightarrow x^2-x\left(6-x\right)+\left(6-x\right)^2-3x-3\left(6-x\right)=-6\)

\(\Leftrightarrow x^2-6x+8=0\)

\(\Leftrightarrow\left(x-4\right)\left(x-2\right)=0\)

\(\Leftrightarrow\left[{}\begin{matrix}x=4\left(tm\right)\Leftrightarrow y=2\left(tm\right)\\x=2\left(tm\right)\Leftrightarrow y=4\left(tm\right)\end{matrix}\right.\)

TH2: \(\left\{{}\begin{matrix}x+y+3=27\\\left(x-y\right)^2+\left(x-3\right)^2+\left(y-3\right)^2=2\end{matrix}\right.\)

\(\Leftrightarrow\left\{{}\begin{matrix}x+y=24\\x^2-xy+y^2-3x-3y=-8\end{matrix}\right.\)

\(\Leftrightarrow x^2-x\left(24-x\right)+\left(24-x\right)^2-3x-3\left(24-x\right)=-8\)

\(\Leftrightarrow3x^2-72x+512=0\) (vô nghiệm)

KL: Vậy phương trình có tập nghiệm (x;y) = [(2;4);(4;2)]

 

Bình luận (0)
Aeris
Xem chi tiết
Yim Yim
Xem chi tiết
Hoàng Anh Thắng
Xem chi tiết
Nguyễn Việt Lâm
14 tháng 3 2022 lúc 10:43

- Với \(x=1\) ko thỏa mãn

- Với \(x=2\Rightarrow\dfrac{2}{2y+2}\in Z\Rightarrow\dfrac{1}{y+1}\in Z\Rightarrow y=\left\{-2;0\right\}\) ko thỏa mãn

- Với \(x\ge3\)

\(x^2-2⋮xy+2\Rightarrow x\left(xy+2\right)-y\left(x^2-2\right)⋮xy+2\)

\(\Rightarrow2\left(x+y\right)⋮xy+2\)

\(\Rightarrow\left(x-2\right)\left(y-2\right)\le2\)

\(\Rightarrow y-2\le\dfrac{2}{x-2}\le\dfrac{2}{3-2}=2\Rightarrow y\le4\)

\(\Rightarrow y=\left\{1;2;3;4\right\}\)

Lần lượt thay 3 giá trị của y vào pt biểu thức ban đầu

Ví dụ: \(y=1\Rightarrow\dfrac{x^2-2}{x+2}\in Z\Rightarrow x-2+\dfrac{2}{x+2}\in Z\)

\(\Rightarrow x+2=Ư\left(2\right)\Rightarrow\) ko tồn tại x nguyên dương t/m

Tương tự...

Bình luận (4)
Trương Mỹ Hoa
Xem chi tiết